Chap 7 - Standard Costing

February 13, 2018 | Author: Erica Uy | Category: Debits And Credits, Variance, Self-Improvement, Motivation, Production Function
Share Embed Donate


Short Description

Agamata Management Advisory Services Chapter 7 testbank...

Description

Chapter 7

Standard costing and variance analysis

254

MULTIPLE CHOICE Basic concepts 1. Which one of the following is true concerning standard costs? A. Standard costs are estimates of costs attainable only under the most ideal conditions, but rarely practicable. B. Standard costs are difficult to use with a process costing system. C. If properly used, standards can help motivate employees. D. Unfavorable variance, material in amount, should be investigated, but large favorable variance need not be investigated. (aicpa) 1. C ? A true statement concerning standard costs.  Standard costs are established in business to serve as a reasonable and reliable basis in budgeting and performance evaluation. Standard costs should be established with the knowledge and participation of employees to foster motivation, to encourage goal congruence, extend necessary efforts, and produce expected results; necessary efforts, choice-letter “c” is correct. Standard costs are also used to determine and investigate variances, both material and immaterial in figure. Choice-letter “a” is incorrect because standard costs are adjustable to actual operating conditions and are normally used in practice. Choice-letter “b” is incorrect because standard costs are easier to apply in process costing where products are produced in a homogeneous fashion. Choice-letter “d” is also incorrect because variances derived using standard costs should be investigated, studied, and explained of regardless of the size of the variance. Also, standard costing is used to activate necessary remedies, institute corrective actions and foster greater efficiencies. 2. Which of the following is a purpose of standard costing? A. Determine breakeven production level. B. Control costs. C. Eliminate the need for subjective decisions by management. D. Allocate cost with more accuracy.

(aicpa)

2. B ? A purpose of standard costing.  Standard costing is established to predict normal performance, set the operational efficiency level, inform employees of their expected output, guide and direct employees performances, serve as a reliable and reasonable basis in planning and budgeting, help in setting managerial goals, serve as a basis in evaluating actual results, and is used as a benchmark for strategic improvements. Standard costs greatly help in controlling costs. Choice-letter “b” is correct. Choice-letter ‘a” is not the best answer although standard costs are used in the determination of breakeven point, however, it is not a primary purpose of standard costing. Choice-letter “c” is incorrect because standard costing does not eliminate the subjective decisions of management but rather assists in providing informed decisions. Choice-letter “d” is incorrect because standard cost is not primarily used

Chapter 7

Standard costing and variance analysis

255

to allocate costs, and therefore, if ever used shall not be an accurate model of allocating costs. 3. A standard cost system may be used in A. Job-order costing but not process costing. B. Either job-order costing or process costing. C. Process costing but not job-order costing. D. Neither process costing nor job-order costing.

(aicpa)

3. B ? The use standard cost system in relation to job-order and process costing method.  Standard cost system is developed to complement other accounting cost systems such as job-order costing and process costing, choice-letter “b” is correct. It could also be used in relation with the newly developed costing systems such as activity-based costing, life-cycle costing, and backflush costing. Standard costs are meant to guide and predict performances and are greatly related and interrelated with other managerial techniques, including that of costing systems. 4. When a manager is concerned with monitoring total cost, total revenue, and net profit conditioned upon the level of productivity, an accountant should normally recommend Flexible Budgeting Standard Costing A. Yes Yes B. Yes No C. No Yes D. No No (aicpa) 4. A ? An accountant’s normal recommendation when a manager is concerned with monitoring revenues, costs and profit conditioned upon the level of productivity.  Flexible budgeting is the best budgetary model to be used when different levels of activity are considered. Standard costing, which sets expected revenues and costs, serves as value-added technique to budgeting both in the area of planning and controlling.(i.e., performance monitoring and evaluation). Flexible budgeting and standard costing are managerial techniques used to predict, monitor, and evaluate results, and to motivate personnel in achieving targets, improving systems, and produce excellent results. 5.

The absolute minimum cost that would be possible under the best operating conditions is a description of which type of standard cost? A. Currently attainable (expected). B. Theoretical. C. Normal. D. Practical. (aicpa)

5. B ? The standard of absolute minimum cost that would be possible under the best operating condition.

Chapter 7



Standard costing and variance analysis

256

Choice-letter “b” is correct. A theoretical standard cost is the absolute minimum cost that would be possible under the best conceivable operating conditions. Choiceletter “a” is incorrect because currently attainable (expected or budgeted) costs is the target cost that employees are expected to attain under an efficient conditions. Choice-letter “c” is incorrect because normal capacity is the long-term average unit of production used in the calculation of fixed overhead rate and does not indicate best operating conditions. Choice-letter “d” is also incorrect because practical capacity indicates the most realistic level of production performance given that the conditions shall be unchanged, and therefore does not refer to the best operating performance.

6. Management scrutinizes variances because A. Management desires to detect such variances to be able to plan for promotions. B. Management needs to determine the benefits forgone by such variances. C. It is desirable under conventional knowledge on good management. D. Management recognizes the need to know why variances happen to be able to make corrective actions and fairly reward good performers. (rpcpa) 6. D ? A managerial reason for variance analysis.  Variance analysis is done to know when did the variance happen, where does the variance occur, how much is the variance, how does it happen, and why is there a variance to make immediate corrective measures, if necessary, Variances are also determined to reward good performances (choice letter “d” is correct). Choice-letters “a”, “b”, and “c” are only incidental and complementary uses of variance analysis. Choice-letter “a” is incorrect because variance analysis is not primarily geared towards giving promotions to top performers but rather to monitor and check results. Giving of promotions is part of the reward system and is under the recognition policies, which relates to motivation. Motivation is also the primary aim of standard costing but the reward system may vary from a firm to another. Choice-letter “b” is incorrect because the benefits foregone by a variance are not much of value to management but more on how to prevent error occurrences and improving performances. Choice-letter “c” is incorrect because the given choice does not provide a concrete and a valid rationale for scrutinizing variances as offered by other choices. 7. Which of the following factors should not be considered when deciding whether to investigate a variance? A. Magnitude of the variance and the costs of investigation. B. Trend of the variance over time. C. Likelihood that an investigation will eliminate future occurrences of the variance. D. Whether the variance is favorable or unfavorable. (cia) 7. D ? A factor not considered in deciding whether to investigate a variance.  A variance should be investigated – measured, studied, researched, explained of and given appropriate action to improve situation and avoid recurrence of the same in the future. In investigating a variance, the costs involved should be considered in relation to the benefit that could be derived in knowing and understanding a variance.

Chapter 7

Standard costing and variance analysis

257

The amount, materiality, or magnitude of a variance, together with its trends are factors considered in investigating a variance. Choice-letters “a”, “b”, and “c” are statements describing factors that are considered in deciding whether to investigate a variance. Choice-letter “d” is the correct answer because it is an incorrect statement. A variance should be investigated, whether it is favorable or unfavorable. 8. A difference between standard costs used for cost control and the budgeted costs of the same manufacturing effort A. Can exist because standard costs represent what costs should be whereas budgeted costs are expected actual costs. B. Can exist because budgeted costs are historical costs, whereas standard costs are based on engineering studies. C. Can exist because budgeted costs include some slack, whereas standard costs do not. D. Cannot exist because the amounts should be the same. (cma) 8. A ? A difference between standard costs and budgeted costs.  Standard costs refer to the estimated costs based on actual capacity, or the “should be” costs. Budgeted costs are estimated costs on budgeted capacity or “ought to be” costs. Budgeted costs are expected actual costs. Choice-letter “a” is correct. Standard costs and budgeted costs differ in the short-run. Choice-letter “b” is incorrect because budgeted costs are not historical costs. Choice-letter “c’ is incorrect because slack (or the practice of overestimating expenses) is not encouraged either in setting standard costs and budgeted figures. Choice-letter “d” is incorrect since there is a difference between standard costs and budgeted costs in the short-run, although they may have no difference in the longrun. 9. The best basis upon which standard cost should be set to measure controllable production inefficiencies is A. Engineering standards based on ideal performance. B. Normal capacity. C. Engineering standards based on attainable performance. D. Practical capacity. (cma) 9. C ? The best basis upon which standard cost should be set.  Standard costs should be measurable, realistic and attainable. Measurable to be used in performance evaluation. Realistic to conform with the existing conditions of people, machine, systems, and other strategic factors of production. Attainable to give motivation for achievement. Standards are not primarily set based on historical or past performance. Standards are more effective if based on upcoming conditions and resources. Choice-letter “c” is correct. Choice-letter “a” is incorrect because standards on ideal performance may demotivate other employees especially those who, at their best, produce average results. Choice-letter “b” is incorrect because normal capacity does not reflect the

Chapter 7

Standard costing and variance analysis

258

seasonal or periodical variations in production and therefore would not give the best basis in measuring production inefficiencies. Choice-letter “d” is incorrect because practical capacity (i.e., 100% less normal allowances for stoppages, employee inefficiencies, etc.) though realistic, however, does not unfold better results, does not harness organizational potentials, and does not institute systems improvements. 10. Which one of the following statements about ideal standards is incorrect? A. Ideal standards are also called theoretical or maximum-efficiency standards. B. Ideal standards do not make provisions for workers with different degrees of experience and skill levels. C. Ideal standards make no allowance for waste spoilage, and machine breakdowns. D. Ideal standards can be used for cash budgeting or product costing (cma) 10. D ? An incorrect statement about ideal standards.  Ideal (i.e., theoretical, maximum-efficiency) standards represent the best performance of the organization. It does not give allowances for errors, inefficiencies, waste, spoilage, machine breakdowns, and downtime. It is a standard set for production under optimal conditions. Ideal standards can have motivational effects if workers are primed and directed to aspire for better performance and strive for excellence. Otherwise, ideal standards may have negative effects to the employee performances. Ideal standards are used under the total quality management environment where search for continuous improvement is always ongoing and errors and inefficiencies are identified for elimination. However, ideal standards are ordinarily replaced by currently attainable standards for cash budgeting, product costing, and in estimating departmental performance. This is make reasonable financial planning possible. Choice-letters “a”, “b”, and “c” are correct statements about ideal standards. 11. Which one of the following statements pertaining to practical standards is incorrect? A. Practical standards can be used for product costing and cash budgeting. B. A firm using practical standards has no reason to make any midyear adjustment to the production standard if an old machine is replaced by a newer, faster machine. C. Under practical standard, exceptions from standards are less likely, consequently, managers will be better able to practice management by exception. D. Practical standard are more likely to be attained by worker making diligent efforts.

(cma)

11. B ? An incorrect statement about practical standards.  Practical standards are currently attainable level of performance after giving effect to normal allowances for spoilage, inefficiencies, machine breakdowns, production stoppages, and the like. In the absence of better standard, practical standard may be used for product costing and, therefore, budgeting. Consequently, variances may be derived and analyses are to made. Since practical standards provide allowances for normal production interruptions and inefficiencies, diligent workers are likely to meet

Chapter 7

Standard costing and variance analysis

259

this benchmark. Choice-letter “a”, “c”, and “d” are correct statements with regard to practical standards. Choice-letter “b” is an incorrect statement because practical standards are not designed to be inflexible without regard to major changes in the methods, techniques, and systems employed by an organizational. Hence, choice-letter “b” is the correct answer. 12. Which of the following cost allocation methods would be used to determine the lowest price that could be quoted for a special order that would utilize the capacity within a production area. A. Job order. C. Variable. B. Standard. D. Process. (rpcpa) 12. C ? The cost allocation method that would be used to determine the lowest price that could be quoted for special order.  A special order situation is normally entertained when there is an available capacity, the regular business is not disturbed, and there is an absence of competition that may arise resulting from accepting the special sales order. Given this environment, special sales order is normally priced lower than the regular sales. However, its price should not be lower than the incremental cost of accepting the order. This accounting calls for the application of incremental costing (or differential costing, marginal costing, variable costing). Hence, choice-letter “c” is correct. Choice-letters “a” and “d” are incorrect because these methods do not segregate variable costs from fixed costs. Choice-letter “b” is an inferior choice because standard costing relates to normal operations and not in determining the lowest price that could be quoted for a special order. 13. If the total materials variance (actual cost of materials used compared with the standard cost of the standard amount of materials required) for a given operation is favorable, why must this variance be further evaluated as to a price and usage? A. There is no need to further evaluate the total materials variance if it is favorable. B. Generally accepted accounting principles require that all variances be analyzed in three stages. C. All variances must appear in the annual report to equity owners for proper disclosure. D. Determining price and usage variance allows management to evaluate the efficiency of the purchasing and production functions. 13. D ? The reasons why a favorable materials variance should still be evaluated.  A variance (difference between actual and standard costs) always indicates an error. An error (i.e., variance) whether good (favorable) or bad (unfavorable) should be investigated to know, understand, and control its recurrence. Managers, though accepting a degree of flexibility in their plans, ought to use talents, methods, organizational skills, techniques, human relations, and other scientific and relational approaches to make things happen as expected. Plans are very important guideposts in managing. If plans are not met, variances should be studied to

Chapter 7

Standard costing and variance analysis

260

improve systems, incorporate variables, adjust plans, or change to make the plan as accurate and reliable as possible and manage the process in conformance with the plans. Analyzing the details of a favorable materials variance may result to relevant information useful in directing the operational affairs of an entity. Choice-letter “d” is correct, determining price and usage variance allows management to evaluate the efficiency of the purchasing and production functions. Choice-letter “a” is incorrect because a favorable variance still needs to be evaluated. Choice-letter “b” is incorrect because generally accepted accounting principles are not concerned with variances and their analyses. Choice-letter “c” is incorrect because variances are not required to appear on a general purpose financial report since variances are primarily designed for management use and disposition only. 14.When standard costs are used in a process costing system, how, if at all, are equivalent units of production (EUP) involved or used in the cost report at standard? A. Equivalent units are not used. B. Equivalent units are computed using a special approach. C. The actual equivalent units are multiplied by the standard cost per unit. D. The standard equivalent units are multiplied by the actual cost per unit. (aicpa) 14. C ? The use of equivalent unit of production in the cost of production report using process costing system.  Equivalent units of production measures the theoretical units produced in a given period. It is a more accurate basis in determining the cost of production, instead of using the actual units produced where a portion of such may have been produced last period and some units may not have been actually completed although a portion of which have already been produced. Equivalent unit of production (or equivalent production) substitutes the actual units of production in the cost of production report. It is therefore the units multiplied by the standard unit cost in determining the costs to be accounted. Choice-letter “c” is correct. Choice-letter “a” is incorrect because equivalent unit is used in the cost of production report. Choice-letter “b” is incorrect because there is no need for a special approach in determining equivalent units but only to consider the work-inprocess inventories and their degree of completion. Choice-letter “d” is incorrect because the standard equivalent units of production is multiplied by the standard, not actual, cost per unit. 15 When items are transferred from stores to production, an accountant debits work-inprocess and credits material accounts. During production, a materials quantity variance may occur. The materials quantity variance is debited for an unfavorable variance and credited for a favorable variance. The intent of variance entries is to provide A. Accountability for materials lost during production. B. A means of safeguarding assets in the custody of the system. C. Compliance with GAAP. D. Information for use in controlling the cost of production. (cia)

Chapter 7

Standard costing and variance analysis

261

15. D ? The intent of the materials variance entries.  Choice-letter “d” is correct. Variances ought to be measured as they occur or at the earliest possible time. This will provide managers a timely opportunity to avert unnecessary accumulation of variances due to delayed reporting. Hence, one of the reasons information on variances are needed is to control cost of production Choice-letter “a” is incorrect because variances do not by themselves define accountability of costs and lost units which are rightly determined when responsibility centers are established or when organizational lines and scope of authority and responsibility were defined. Choice-letter “b” is incorrect because variances are reflections of actual and standard costs, which are directly related to recording and reporting systems and not on the physical custodianship of assets. Choice-letter “c” is incorrect because variances and variance analyses are not governed by the generally accepted accounting principles. 16. The difference between the actual amounts and the flexible budget amounts for the actual output achieved is the A. Production volume variance. C. Sales volume variance. B. Flexible budget variance. D. Standard cost variance. (cma) 16. B ? The difference between the actual amounts and the flexible budget amounts.  A flexible budget is an estimate based on actual capacity. The difference between the actual costs and flexible budget is called flexible budget variance. Choice-letter “b” is correct. Choice-letter “a” is incorrect because production volume variance is the difference between normal capacity (or if not available, the budgeted capacity) and actual capacity. Choice-letter “c” is incorrect because sales volume variance is the difference between actual sales quantity and budgeted sales quantity. Choice-letter “d” is incorrect because standard cost variance is the difference between actual cost and standard cost. 17. The flexible budget variance in operating income is A. Actual operating income minus flexible budget operating income. B. Budgeted unit price times the difference between actual inputs and budgeted inputs for the actual activity levels achieved. C. A flexible budget amount minus a static budget amount. D. Actual unit price minus budgeted unit price times the actual units produced. (adapted) 17. A ? A statement regarding flexible-budget variance in operating income.  Flexible budget variance is the difference between actual costs and flexible budget. Flexible budget variance in operating income is the difference between actual operating income and flexible budget operating income. Choice-letter “a” is correct.

Chapter 7

Standard costing and variance analysis

262

Choice-letter “b” is incorrect because it pertains to sales quantity variance. Choice-letter “c” is incorrect because it refers to activity level (or production level ). Choice-letter is “d” is incorrect because it refers to price variance. 18. An efficiency variance equals A. A flexible budget amount minus a static budget amount. B. Actual operating income minus flexible budget operating income. C. Actual unit price minus budgeted unit price, times the actual units produced. D. Budgeted unit price times the difference between actual inputs and budgeted inputs for the actual activity level achieved. 18. D ? A statement regarding efficiency variance.  Efficiency refers to hours or the ability to make optimum use of time. Budgeted inputs on actual activity levels means standard hours on actual activity. The difference between actual hours (i.e., actual inputs) and standard hours is the efficiency variance. Choice-letter “a” is incorrect because it is an activity level variance. Choice-letter “b: is incorrect because it refers to flexible operating income variance. Choice-letter “c” is incorrect because it refers to production price variance. 19. The budget for a given cost during a given period was P80,000. The actual cost for the period was P72,000. Considering these facts, the plant manager has done a better-than-expected job in controlling the cost if A. The cost is variable and actual production was 90% of budgeted production. B. The cost is variable and actual production equaled budgeted production. C. The cost is variable and actual production was 80% of budgeted production. D. The cost is a discretionary fixed cost and actual production equaled budgeted production. (aicpa) 19. B ? The circumstance that a plant manager had done a better-than-expected job.  The plant manager did a better-than-expected job when production costs have scaled down with a parallel increase in or equal level of production units, or when the variance is favorable. Choice-letter “b” is the best answer. Choice-letter “a” is incorrect because given a 90% actual level of capacity, the standard costs shall be P72,000 (i.e., P80,000 x 90%) which equals the actual cost, and therefore, there is no cost variance. Choice-letter “c” is incorrect because there would be an unfavorable variance in the amount of P8,000 based on standard cost of P64,000 (i.e., P80,000 x 80%). Choice-letter “d” is incorrect because it is not clear whether the discretionary fixed cost is controlled by the plant manager. Besides, the material fact is the change in production level and it relates to variable cost and not much to total discretionary fixed costs. Materials cost variances 20.Which department is customarily held responsible for an unfavorable materials usage variance? A. Quality control. C. Engineering.

Chapter 7 B. Purchasing.

Standard costing and variance analysis D. Production.

263 (aicpa)

20. D ? The department primarily responsible for an unfavorable materials usage variance.  Materials usage variance is the difference between actual quantity used and standard quantity that should have been used given the level of actual production. The standard-setting team of which the production manager is a member thereof determines standard quantity per unit of output. The production manager should be the primary officer that will play a key role in establishing the quantity of materials needed per finished product. A usage variance could be precipitated by allocation of skilled and non-skilled workers, production stoppages, and repairs. Workers are supposed to have passed the rigidity of human resource requirements, in terms of experience, training, and skills. Materials are supposed to have passed the quality requirement. At the end, it is still the production manager that could be held primarily responsible for a materials usage variance. Choice-letter “d” is correct. Choice-letter “a” is incorrect because quality control manager is responsible for a plant wide quality standards and not only for materials usage during production. Choice-letter “b” is incorrect because purchasing manager is primarily responsible for the materials price variance. Choice-letter “c” is incorrect because engineering manager is responsible for design, engineering, and related quality standards. 21. The standard unit cost is used in the calculation of which of the following variances? Materials Price Materials Usage Variance Variance A. No No B. No Yes C. Yes No D. Yes Yes (aicpa) 21. D ? The variances that use standard unit costs.  Materials price variance is the difference in actual and standard prices multiplied by actual quantity. Materials quantity variance is difference in actual and standard quantity times standard unit cost. Both of these materials variances use standard unit cost. 22. Under a standard cost system, the materials price variances are usually the responsibility of the A. Production manager. C. Sales manager. B. Cost accounting manager. D. Purchasing manager. (cma) 22. D ? The manager that is usually held accountable for materials price variance.  Materials price variance, which is the difference between the actual price and the standard price, is primarily the accountability of the purchasing manager. A member of the standard-setting team, the purchasing manager acts as a lead officer in establishing expected materials prices, after considering upcoming possibilities that may affect prices. Once established, the purchasing manager should always be on

Chapter 7

Standard costing and variance analysis

264

the lookout, vigilant, with the movements in materials prices, either economic-driven, industry-driven, or supplier-driven movements. Choice-letter “d” is correct. Choice-letter “a” is incorrect because production manager is primarily responsible with the materials usage. Choice-letters “b” and “c” are incorrect because cost accounting manager and sales manager have nothing to do with materials prices. 23. If a company follows a practice of isolating variances as soon as possible, the appropriate time to isolate and recognize a direct materials price variance is when. A. Materials are issued. B. Materials are purchased. C. Materials are used in production. D. The purchase order originates. (aicpa) 23. B ? The appropriate time to isolate materials price variance.  Choice-letter “b” is correct. Variances should be identified, recorded, and corrected at the earliest possible time. Materials price variances should be identified on the date of purchase, being the earliest possible time to isolate the price variance. Choice-letters “a” and “c” are incorrect because isolating materials variances on the date materials are issued to production will unduly accumulate unnecessary price variances that could have been corrected prior to issuance of materials to production. Choice-letter “d” is incorrect because materials price variance cannot still be identified in as much as there is no actual price yet.. 24. An unfavorable price variance occurs because of A. Price increases for raw materials. B. Price decreases for raw materials. C. Less-than-anticipated levels of waste in the manufacturing process. D. More-than-anticipated levels of waste in the manufacturing process.

(aicpa)

24. A ? The reason an unfavorable price variance occurs.  An unfavorable price variance occurs when the actual price is greater than the standard price of production. Choice-letter “b” is incorrect because it will result to favorable price variance. Choice-letter “c’ is incorrect because it will result to a favorable usage variance. Choice-letter “d” is also incorrect because it will result to an unfavorable usage variance. 25. The Purchasing Manager of XY Company decided to buy 65,000 bags of flour with a quality rating two grades below that which the company normally purchased. This purchase covered about 90% of the flour requirements for the period. As to the material variances, what will be the likely effect? A. Unfavorable price variance, favorable usage variance. B. Favorable price variance, unfavorable usage variance. C. No effect on price variance, unfavorable price variance. D. Favorable price variance, favorable usage variance. (rpcpa)

Chapter 7

Standard costing and variance analysis

265

25. B ? The likely effect to material variances of buying a material two grades below which the company normally purchased.  A two grade below quality of materials would rationally mean a lower purchase price resulting to favorable price variance. An inferior quality of materials would also naturally translate to more quantity of materials used due to breakages, unsuitability of materials in the operations, and unfamiliar handling of production personnel resulting to more wastages, etc. Choice-letter “b” is the best logical answer. 26. In a standard cost system, the materials price variance is obtained by multiplying the A. Actual price by the difference between actual quantity purchased and standard quantity used. B. Actual quantity purchased by the difference between actual price and standard price. C. Standard price by the difference between standard quantity purchased and standard quantity used. D. Standard quantity purchased by the difference between actual price and standard price. (aicpa) 26. B ? The process of computing the materials price variance.  Materials price variance is the difference in actual price and standard price, multiplied by the actual quantity purchased. Choice-letter “b” is correct. Choice-letters “a” and “c” are incorrect because the computations do not result to any relevant variance. Choice-letter “d” is incorrect because materials price variance is normally multiplied by the actual quantity purchased, not standard quantity purchased. 27. Under a standard cost system, the materials efficiency variances are the responsibility of A. Production and industrial engineering. B. Purchasing and industrial engineering. C. Purchasing and sales. D. Sales and industrial engineering. (cma) 27. A ? The office that is responsible for materials efficiency variances.  Materials efficiency (or usage) variance arises from improper use of materials, wastage, spoilage, shrinkage, or theft. It is primarily a responsibility of the production department. The industrial engineering department may have a responsibility in the materials usage variance inasmuch it plays a role in the establishment of standard quantity and in the design of the production process. Choice-letters “b”, “’c”, and “d” are incorrect because purchasing and sales departments have no roles in the design of the production process and in the establishment of the standard materials usage, and in the use of materials.

Chapter 7

Standard costing and variance analysis

266

28. Price variances and efficiency variances can be key to the performance measurement within a company. In evaluating the performance within a company, material efficiency variance can be caused by all of the following except the A. Performance of the workers using the material. B. Actions of the purchasing department. C. Design of the product. D. Sales volume of the product. (cma) 28. D ? The item that does not cause material efficiency variance.  Materials usage (or quantity) variance is caused by many factors such as improper assignment of skilled or non-skilled manpower, defective design of the production process, ineffective maintenance of equipment, and even lower-than-standard-quality of materials. Choice-letters “a”, “b”, and “c” are factors causing materials efficiency variance, and, therefore are incorrect answers. Choice-letter “d”, sales volume, has nothing to do with materials usage variance and is the correct answer. 29. A favorable materials price variance coupled with an unfavorable materials usage variance would most likely result from A. Machine efficiency problems. B. Product mix production changes. C. Labor efficiency problems. D. The purchase of lower-than-standard-quality materials. (cma) 29. D ? The cause of a favorable materials price variance and unfavorable materials usage variance.  A favorable materials price variance indicates savings arising from lower purchase price. A reduction in materials price may be caused by inferior quality resulting to more spoilages, wastages, and other production interruptions. Choice-letter “d” is correct. Choice-letters “a”, “b”, and “c” are incorrect because machine efficiency problems, product mix production changes, and labor efficiency problems have nothing to do with materials purchase price variance. 30. Troop Company had budgeted 50,000 units of output using 50,000 units of raw materials at a total material cost of P100,000. Actual output was 50,000 units of product requiring 45,000 units of raw materials at a cost of P2.10 per unit. The direct material price variance and usage variance were: Price Usage A. P 4,500 unfavorable P10,000 favorable B. P 5,000 favorable P10,500 unfavorable C. P 5,000 unfavorable P10,500 favorable D. P10,000 favorable P 4,500 unfavorable (aicpa) 30. A ? The direct material price and usage variances.

Chapter 7



Standard costing and variance analysis

267

Direct material price variance is the difference in actual price and standard price, multiplied by actual quantity. Standard price per unit is P2.00 (i.e., P100,000 / 50,000 units). The materials price variance is computed as: MPV = (AP – SP) x AQ = (P2.10 – P2.00) x 45,000 units = P4,500 UF Direct materials usage variance is the difference in actual quantity and standard quantity, times standard materials price per unit. The standard materials per unit is 1 (i.e., 50,000 materials / 50,000 units). Hence, the standard quantity for 50,000 units of output is 50,000 materials (i.e., 50,000 units x 1 material). The direct materials usage variance is determined as follows: MQV = (45,00 units – 50,000 units) P2 = (5,000)F x P2 = P(10,000) F

31. Information on Dean Company’s direct-material costs for the month of January 2006 was as follows: Actual quantity purchased 18,000 Actual unit purchase price P 3.60 Materials purchase price varianceunfavorable (based on purchases) P 3,600 Standard quantity allowed for actual production 16,000 Actual quantity used 15,000 For January 2006 there was a favorable direct material usage variance of A. P3,360 C. P3,400 B. P3,375 D. P3,800 (aicpa) 31. C ? The favorable direct usage variance.  Direct usage variance (or quantity variance) is the difference in actual quantity and standard quantity, multiplied by standard unit cost. The standard unit cost is not given, and should be derived from materials purchase price variance as follows: Mat purchase price variance = (Actual price – Standard price) x Actual quantity purchased 3,600 = (P3.60 – SP) x 18,000 units 3,600 = 64,800 – 18,000SP SP = 61,200 / 18,000 = P3.40

Alternatively, the standard purchase price variance may be determined as follows: Actual purchase price P3.60 Change in price (P3,600 UF / 18,000) 0.20 UF Standard purchase price P3.40 The change in unfavorable purchase price is deducted from the actual price because it is supposed to be higher than the standard price. Now that standard unit cost is derived, the materials usage variance shall be computed as follows: Mat usage variance = (Actual quantity used – Standard quantity) x Standard unit cost

MUV = (15,000 – 16,000) x P3.40 MUV = (1,000) F x P3.40 = (P3,400) F There is a favorable quantity variance because actual quantity is smaller than standard quantity.

Chapter 7

Standard costing and variance analysis

32. Information on Kenon Company’s direct material costs is as follows: Standard unit price P 3.60 Actual quantity purchased 1,600 Standard quantity allowed for actual production 1,450 Materials purchase price variance – favorable P 240 What was the actual purchase price per unit, rounded to the nearest cent? A. P 3.00 C. P 3.45 B. P 3.11 D. P 3.75

268

(aicpa)

32. C ? Actual materials purchase price per unit.  By using the materials purchase price variance formula, we can derived the actual materials purchase price per unit, as follows: Materials purchase price variance = (AP- SP) x AQ purchased (240) F (240) 5,760 – 240 AP

= = = =

(AP – P3.60) x 1,600 1,600AP - 5,760 1,600AP 5,520 / 1,600 = P3.45

Alternatively, the actual materials purchase price per unit is: Standard price per unit P 3.60 Price variance per unit (P240 / 1,600) (0.15) F Actual price per unit P 3.45 33. Information on Rex Company’s direct material costs for May 2006 is as follows: Actual quantity of direct materials purchased and used 30,000 lbs. Actual cost of direct materials P 84,000 Unfavorable direct materials usage variance P 3,000 Standard quantity of direct materials allowed for May production 29,000 lbs. For the month of May, what was Rex’s direct materials price variance? A. P2,800 favorable. C. P6,000 unfavorable. B. P2,800 unfavorable. D. P6,000 favorable. (aicpa) 33. D ? Direct materials price variance.  Materials price variance (MPV) is the difference in actual price and standard price, multiplied by actual quantity purchased. The actual price is P2.80 (i.e., P84,000/30,000). The standard price is not given but could be derived from the materials usage (quantity) variance as follows: Materials usage variance P3,000 UF P3,000 SP

= = = =

(AQ – SQ) x Standard price (30,000 – 29,000) x SP 1,000 SP 3,000 / 1,000 = P3.00

Given a standard price of P3.00, the materials price variance (MPV) may now be computed as: MPV = (AP – SP) x AQ purchased MPV = (P2.80 – P3.00) x 30,000 units MPV = (P0.20)F x 30,000 units = (P6,000)F

Chapter 7

Standard costing and variance analysis

269

34.Hankies Unlimited has a signature scarf for ladies that is very popular. Certain production and marketing data are indicated below: Cost per yard of cloth P 36.00 Allowance for rejected scarf 5% of production Yards of cloth needed per scarf 0.475 yards Airfreight from supplier P0.60/yard Motor freight to customers P0.90/scarf Purchase discount from supplier 3% Sales discount to customers 2% The allowance for rejected scarf is not part of the 0.475 yard of cloth per scarf. Rejects have no market value. Materials are used at the start of production. Calculate the standard cost of cloth per scarf that Hankies Unlimited should use in its cost sheets A. P16.87 C. P18.21 B. P17.76 D. P17.30 (rpcpa) 34. B ? The amount of standard cost of clothe per scarf.  The standard cost of materials is composed of the net invoice price plus all necessary costs of handling the materials purchased. The materials input per unit of scarf is 0.50 yard (i.e., 0.475 yd./95%). This is because of the adjustment made on the allowance for rejected scarf. Given this input base, the standard cost of materials shall be: Net purchase price (P36 per yd x 0.50 yd. X 97%) P17.46 Airfreight from supplier/freight-in (P0.60 x 0.50) 0.30 Standard cost of materials per scarf P17.76 Questions 35 through 37 are based on the following information. Burger Queen uses a standard costing system in the manufacture of its single product. The 35,000 units of raw material in inventory were purchased for P105,000, and two units of raw materials are required to produce one unit of final product. In November, the company produced 12,000 units of product. The standard allowed for material was P60,000, and there was an unfavorable quantity variance of P2,500. 35. Burger Queen’s standard price for one unit of materials is A. P2.50 C. P5.00 B. P3.00 D. P6.00

(cma)

35. A ? The standard price per unit of material.  The standard quantity per finished good is 2 units, and the total standard quantity is 24,000 units (i.e., 12,000 units x 2). The standard price is computed by dividing standard costs by standard quantity, or P2.50 (i.e., P60,000 / 24,000 units). 36. The units of material used to produce November output totaled

Chapter 7

Standard costing and variance analysis

A. 12,000 units. B. 23,000 units.

C. 24,000 units. D. 25,000 units.

270

(cma)

36. D ? The actual materials quantity used in November.  The actual quantity used in November is Standard quantity 24,000 units Quantity variance (P2,500 /P2.50) 1,000 UF Actual quantity 25,000 units The actual quantity variance may also be derived from the materials quantity variance as follows: Materials quantity variance 2,500 UF 2,500 AQ

= = = =

(AQ – SQ) x Standard price (AQ – 24,000) x P2.50 2.50AQ – 60,000 62,500 / 2.5 = 25,000 units

37. The materials price variance for the units used in November was A. P 2,500 unfavorable. C. P12,500 unfavorable. B. P15,000 unfavorable. D. P 2,500 favorable.

(cma)

37. C ? The materials price variance for the units used.  The actual materials unit price is P3.00 (i.e., P105,000 / 35,000 units). The actual units used and standard price per unit have been determined in the preceding questions. Materials price variance on the units used is the difference in price multiplied by actual quantity used, and is computed as follows: Actual quantity used 25,000 units x Difference in unit price (P3.00 – P2.50) P 0.50 UF Materials price variance P12,500 UF Questions 38 and 39 are based on the following information. A company produces a gasoline additive. The standard costs and input for a 500-liter batch of the additive are represented below. Chemical Echol Protex Benz CT-40

Standard Input Quantity in Liters 200 100 250 50 600

Standard Cost Per Liter P.200 .425 .150 .300

Total Cost P 40.00 42.25 37.50 15.00 P135.00

The quantities purchased and used during the current period are shown below. A total of 140 batches were made during the current period. Chemical Echol Protex Benz

Quantity Purchased (Liters) 25,000 13,000 40,000

Total Purchased Price P 5,365 6,240 5,840

Quantity Used (Liters) 26,600 12,880 37,800

Chapter 7

Standard costing and variance analysis

CT-40 Total

7,500 85,500

2,220 P19,665

271

7,140 84,420

38. What is the material mix variance for this operation? A. P294 favorable. C. P94.50 unfavorable B. P388.50 favorable. D. P219.50 favorable.

(gleim)

38. B ? The material mix variance.  Material mix variance is the difference between the actual mix and the standard mix, multiplied by the actual quantity used. The standard mix is based on standard input as follows: Materials Echol Protex Benz CT-40

Actual mix 26,600 12,880 37,800 7,140 84,420

Actual quantity at Mix variance Standard cost standard mix in units – UF (F) per unit (2/6 x 84,420) 28,140 (1,540) F P0.200 (1/6 x 84,420) 14,070 (1,190) F 0.425 (25/60 x 84,420) 35,175 2,625 UF 0.150 (5/60 x 84,420) 7,035 105 UF 0.300 84,420

Mix variance in pesos – UF(F) P (308.00) F (505.75) F 393.75 UF 31.50 UF P (388.50) F

Alternatively, the materials mix variance may be calculated as follows: Actual quantity used at standard price Echol (26,600 x P0.200) P5,320 Protex (12,880 x P0.425) 5,474 Benz (37,800 x P0.150) 5,670 CT-40 ( 7,140 x P0.300) 2,142 P18,606.00 - Actual quantity at std. materials input cost (84,420 units x P0.225) 18,994.50 Materials mix variance P ( 388.50) F The standard input cost of P0.225 is determined by dividing the budgeted materials cost of P135 by the total standard input quantity of 600 units. 39. What is the materials yield variance for this operation? A. P294.50 favorable. C. P 94.50 unfavorable. B. P388.50 favorable. D. P219.50 favorable.

(gleim)

39. C ? The materials yield variance.  Materials yield variance refers to the difference in actual output and standard (or expected) output given the number of materials used in the production. The yield rate (or productivity rate) on a given materials input is equal to standard output over standard materials input, or 83.3333% (i.e., 500 output / 600 input). The materials yield variance may be determined in three (3) methods, as follows: Unit-based method: Standard output (83.333% x 84,420 liters / 500 liters) 140.70 batches - Actual output 140.00 Yield variance 0.70 UF x Standard output cost (P135 / 1 batch) P 135 per batch

Chapter 7

Standard costing and variance analysis

Yield variance

272

P 94.50 UF

Or; if the standard output cost is expressed per liter, the yield variance shall be determined as follows: Materials-based method: Standard output (83.333% x 84,420 liters) 70,350 liters - Actual output (140 batches x 500 liters) 70,000 Yield variance 350 UF x Standard output cost (P135 /500 liters) P 027 per liter Yield variance P 94.50 UF Alternatively, the yield variance may also be determined as follows: Standard formula: Actual quantity at standard materials input cost (84,420 units x P0.225) P18,994.50 - Actual output at Standard Output Cost (140 batches x P135 per batch) 18,900.00 Yield variance P 94.50 UF 40. The efficiency variance for either labor or materials can be divided into a A. Yield variance and a price variance. B. Volume variance and a mix variance. C. Mix variance and a price variance. D. Yield variance and a mix variance.

(cma)

40. D ? The components of efficiency variance for both labor and materials.  As used in the question, efficiency variance refers to materials quantity variance and labor efficiency variance. Materials quantity variance is composed of materials mix variance and materials yield variance. Labor efficiency variance is composed of labor mix and labor efficiency variances. Choice-letter “d” is correct. Choice-letters “a” and “c” are incorrect because price variance is not a component of efficiency variance. Rather, the total variances of materials or labor are composed of price variance and efficiency variance. Choice-letter “b” is incorrect because volume variance is the same as efficiency variance as applied to materials, and therefore already includes the mix variance. 41. The materials mix variance equals A. (Inputs allowed – Inputs used) x budgeted weighted-average materials unit price for the planned mix. B. (Budgeted weighted-average labor rate for planned mix - Budgeted weightedaverage labor rate for actual mix) x inputs used. C. (Inputs allowed – Inputs used) x budgeted weighted-average labor rate for the planned mix. D. (Budgeted weighted-average materials unit cost for planned mix - Budgeted weighted-average materials unit cost for actual mix) x inputs used. (ing)

41. D ? The equivalence of materials mix variance.

Chapter 7



Standard costing and variance analysis

273

One way to compute the materials mix variance is to get the difference between the weighted average unit costs for planned mix and actual mix, and multiplying it by the actual inputs used. Choice-letter “d” is correct. Choice-letter ‘a” is incorrect because it determines the yield variance. Choiceletter “b” is incorrect because it refers to labor mix variance. Choice-letter “c” is incorrect because it computes the labor yield variance.

42. The materials yield variance equals A. (Inputs allowed –Inputs used) x budgeted weighted-average materials unit price for the planned mix. B. (Budgeted weighted-average labor rate for planned mix –budgeted weightedaverage labor rate for actual mix) x inputs used. C. (Inputs allowed –inputs used) x budgeted weighted-average labor rate for the planned mix. D. (Budgeted weighted-average materials unit cost for planed mix – budgeted weighted average materials unit cost for actual mix) x inputs used. (ing) 42. A ? The equivalence of materials yield variance.  Yield variance is the difference in standard output and actual output, times the standard output cost. Stated differently, yield variance is the difference in inputs allowed (standard input) less inputs used (actual inputs) multiplied by the weighted average materials unit price for the planned mix (standard materials input cost). Choice-letter “a” is correct. Choice-letter “b” is incorrect because it pertains to labor rate variance. Choiceletter “b” is incorrect because it describes the labor yield variance. Choice-letter “d” is incorrect because it refers to materials mix variance. Direct labor cost variances 43. Which of the following is the most probable reason a company would experience an unfavorable labor rate variance and a favorable labor efficiency variance? A. The mix of workers assigned to the particular job was heavily weighted towards the use of highly paid experienced individuals. B. The mix of workers assigned to the particular job was heavily weighted towards the use of new relatively low paid unskilled workers. C. Because of the production schedule workers from other production areas were assigned to assist this particular process. D. Defective materials caused more labor to be used in order to produce a standard unit. (aicpa) 43. A ? The most probable reason for an unfavorable labor rate variance and a favorable labor efficiency variance.  Choice-letter “a” is correct. A highly skilled, experienced, and highly paid personnel used to perform a job that can be done by a less skilled personnel would result to higher wages and unfavorable labor rate variance. However, the same would result to a faster, more efficient and productive performance resulting to a favorable efficiency variance.

Chapter 7

Standard costing and variance analysis

274

Choice-letter ‘b” is incorrect because the assignment of less skilled worker than as required would result to unfavorable labor efficiency variance and favorable labor rate variance. Choice-letter “c” is incorrect because it would not have a material effect on labor variances, it relates to production scheduling which may not translate to changes in rate and efficiency. Choice-letter “d” is incorrect because the use of defective materials may result to unfavorable labor efficiency variance, not labor rate variance. 44. Excess direct labor wages resulting from overtime premium will be disclosed in which type of variance? A. Yield. C. Labor efficiency. B. Quantity. D. Labor rate. (aicpa) 44. D ? The variance that discloses overtime premium.  Overtime premium refers to additional rate paid to personnel over and above their regular rates, and therefore, has a direct relation with the labor rate variance. Choice-letter “d” is correct. Choice-letters “a” and “b” are incorrect because yield variance and quantity variance refer to output variance. Choice-letter “c” is incorrect because efficiency variance refers to change in hours used. 45. A debit balance in the labor efficiency variance indicates that A. Standard hours exceed actual hours. B. Actual hours exceed standard hours. C. Standard rate and standard hours exceed actual rate and actual hours. D. Actual rate and actual hours exceed standard rate and standard hours.

(aicpa)

45. B ? A case that indicates a debit balance in the labor efficiency variance.  A debit variance means an unfavorable variance. An unfavorable labor efficiency variance indicates that actual hours exceed standard hours. Choice-letter “b” is correct. Choice-letters “a” and “c” are incorrect because they result to a favorable or credit variance. Choice-letter “d” is incorrect because it includes both an unfavorable labor rate and labor efficiency variances. 46. An unfavorable direct labor efficiency variance could be caused by a (n) A. Unfavorable variable overhead spending variance. B. Favorable variable overhead spending variance. C. Unfavorable fixed overhead volume variance. D. Unfavorable materials usage variance.

(cma)

46. D ? An item that cause an unfavorable efficiency variance.  An unfavorable efficiency variance springs from improper use of personnel skills and time, undue delays or stoppages in production caused by improperly maintained machines, erroneous design, or inferior quality of materials input. Hence, choice-

Chapter 7

Standard costing and variance analysis

275

letter “d”, unfavorable materials usage variance, also causes an unfavorable labor efficiency variance. Choice-letters “a” and “b” are incorrect because variable overhead spending variance refers to variable overhead rate variance and has no effect on the labor efficiency variance. Choice-letter “d” is incorrect because fixed overhead volume variance is related to fixed overhead and not to labor. 47. Which of the following unfavorable variances is directly affected by the relative position of a production process on a learning curve? A. Material mix. C. Labor rate. B. Materials price. D. Labor efficiency. (aicpa) 47. D ? The unfavorable variance that is directly affected by the relative position of a production process on a learning curve.  Learning curve reflects the increase in the productivity rate of employees as they redo the same job. Effectively, learning curve process would improve labor efficiency as production progresses due to experience. This also suggests that initial production processes would most likely result to unfavorable labor efficiency variance. Choiceletter “d” is correct. Choice-letters “a”, “b”, and “c” are not affected by the learning curve process. Material mix is affected by the way actual inputs are used in the production process. Material price is affected by external influences, which may be beyond the normal control of the organization. Labor rate is affected by the quality and availability of personnel, and general industrial conditions. 48. Under a standard cost system, labor price variances are usually not attributable to A. Union contracts approved before the budgeting cycle. B. Labor rate predictions. C. The assignments of different skill levels of workers than planned. D. The payment of hourly rates instead of prescribed piecework rates. (cma) 48. A ? An item not attributable to labor price variances.  Labor price is affected by the changes in labor rate, predictions in labor rate, assignments of different skill levels of workers than planned, and variation in the mode of paying personnel such as from piecework rate to hourly rate. Choice-letter “a” is the correct answer because union contracts approved before the budgeting cycle should have been incorporated in the plans and are expected to be executed as planned. 49. How is labor rate variance computed? A. The difference between standard and actual rates, times standard hours. B. The difference between standard and actual hours, times actual rate. C. The difference between standard and actual rates, times actual hours. D. The difference between standard and actual hours, times the difference between standard and actual rates. (aicpa)

Chapter 7

Standard costing and variance analysis

276

49. C ? The procedure of computing labor rate variance.  Labor rate variance is the difference between actual labor rate and standard labor rate, times actual hours worked. Choice-letter “c” is correct. Choice-letter “a” is incorrect because it is a labor rate variance using the 3-way variance analysis method. Choice-letter “b” is incorrect because it does not give a relevant information and is not computed as a variance for information purposes. Choice-letter “d” is incorrect because it represents the joint labor rate-efficiency variance computed using the 3-way variance analysis method. 50. The difference between the actual labor rate multiplied by the actual hours worked and the standard labor rate multiplied by the standard labor hours is the A. Total labor variance. C. Labor usage variance. B. Labor rate variance. D. Labor efficiency variance. (aicpa) 50. A ? The difference between the actual labor rate multiplied by the actual hours worked and the standard labor rate multiplied by the standard labor hours.  Actual labor rate times actual hours is actual labor costs. Standard labor rate multiplied by standard labor rate is standard labor costs. The difference between actual labor costs and standard labor costs is the total labor variance. Choice-letter “a” is correct. Choice-letter “b” is incorrect because labor rate variance is the difference in labor rates times actual hours used. Choice-letters “c” and “d” are the same and are incorrect because labor usage variance or labor efficiency variance is the difference in labor hours multiplied by the standard labor rate. 51. Listed below are four names for different kinds of standards associated with a standard cost system. Which one describes the labor costs that should be incurred under efficient operating conditions? A. Ideal. C. Maximum-efficiency. B. Basic. D. Currently attainable. (aicpa) 51. D ? The standard describing the labor cost that should be incurred under efficient operating conditions.  Choice-letter “d” is correct. Currently attainable standard refers to the level of performance that could be attained through an efficient utilization of available resources. It is a difficult standard, but is attainable. Choice-letter “a” is incorrect because ideal standard is attainable under the best operating condition. Choice-letter “b” is incorrect because basic standard is the lowest standard and does not mirror an efficient use of resources. Choice-letter “c” is incorrect because maximum-efficiency standard is similar to ideal standard. 52. The flexible budget for the month of May was for 9,000 units at a direct materials cost of P15 per unit. Direct labor was budgeted at 45 minutes per unit for a total of P81,000. Actual output for the month was 8,500 units with P127,500 in direct materials and P77,775 in direct labor expenses. The direct labor standard of 45

Chapter 7

Standard costing and variance analysis

277

minutes was maintained throughout the month. Variance analysis of the performance for the month of May would show a (n) A. Favorable materials usage variance of P7,500. B. Favorable direct labor efficiency variance of P1,275. C. Unfavorable direct labor efficiency variance of P1,275. (cma) D. Unfavorable direct labor price variance of P1,275. 52. D ? The correct expression for the materials and labor variances.  The direct labor variances are tabulated as follows: Hours Rate per hour Total Cost Actual labor (8,500 x 45/60) 6,375 P12.20 P77,775 - Standard labor (8,500 x 45/60) 6,375 12.00 76,500 Labor variance – UF (F) 0 P 0.20 UF P 1,275 The actual labor rate of P12.20 is computed by P77,775/6,375 hrs. The budgeted hours is 6,750 (i.e., 9,000 units x 45/60) and the standard labor rate of P12.00 is computed by using the budgeted data of P81,000/6,750 hours. The labor rate variance is P1,275 unfavorable (i.e., P0.20 UF x 6,375 hrs.). Choice-letter ‘a” is incorrect because materials usage variance cannot be calculated on the given data. Choice-letters “b” and “c” are incorrect because there is no labor efficiency variance since the labor standard hours are maintained throughout the month. 53. Below are Russel Corporation’s standard costs to produce one concrete table: Direct raw materials 2 kgs.@ P375 per kg Direct labor 30 minutes @ 31.25 per hour In September, Russel produced 250 concrete tables. Five hundred twenty (520) kgs of raw materials were used at a total costs of P193,440. A total of 128 direct labor hours were used at a cost of P4,096. The direct labor rate variance is: A. P22.50 C. P64.75 B. P93.00 D. P96.00 (rpcpa) 53. D ? The direct labor rate variance.  Direct labor rate variance is the difference in actual labor rate and standard labor rate multiplied by the actual hours worked. The actual unit labor rate is P32 per hour (i.e., P4,096 / 128 DLH). Therefore, the direct labor rate variance is: DL Rate Variance = (P32 – P31.25) x 128 hrs. = P0.75 UF x 128 hrs. = P96 UF

54. Francine Company uses a standard cost system. Data relating to direct labor for the month of August 2006 is as follows: Direct labor efficiency variance – favorable P 5,250 Standard labor rate P 7.00 Actual direct labor rate P 7.50 Standard hours allowed for actual production 9,000

Chapter 7

Standard costing and variance analysis

What are the actual hours worked for the month of August 2006? A. 9,750 C. 8,300 B. 8,400 D. 8,250

278

(aicpa)

54. D ? The actual hours for the month of August 2006.  Actual hours may be derived from the labor efficiency variance. Direct labor efficiency variance is the difference between actual hours and standard hours, multiplied by standard labor rate. Equation method: DL efficiency variance (P5,250) F (P5,250) AH

= = = =

(AH – SH) x Standard rate (AH – 9,000) x P7.00 7AH – 63,000 57,750 / 7 = 8,250

Alternatively, actual hours may be determined as follows: Direct method: Standard hours 9,000 - Change in hours (P5,250 / P7) 750 F Actual hours 8,250 The change in hours (i.e., 750 hours) is deducted from standard hours because of the presence of favorable efficiency variance, that is, actual hours are less than standard hours. 55. The information on Donnie Company’s direct labor costs for the month of January 2006 is as follows: Actual direct labor hours 34,500 Standard direct labor hours 35,000 Total direct labor payroll P241,500 Direct labor efficiency variance – favorable P 3,200 What is Donnie’s direct labor rate variance? A. P17,250 unfavorable. C. P21,000 unfavorable. B. P20,700 unfavorable. D. P21,000 favorable. (aicpa) 55. B ? The direct labor rate variance.  Labor rate variance is the difference in labor rate multiplied by the actual hours. The actual labor rate is P7.00 (i.e., P241,500 / 34,500 hrs). The standard labor rate is not readily given but could be derived from the labor efficiency variance, as follows: DL efficiency variance (P3,200) F (P3,200) SR

= = = =

(AH – SH) x Standard rate (34,500 – 35,000) x SR (500) SR P3,200 / 500 = P6.40

The direct labor rate variance may now be computed as follows: DL rate variance = (AR – SR) x Actual hours DL rate variance = (P7.00 – P6.40) x 34,500 hrs. = P20,700 UF

Chapter 7

Standard costing and variance analysis

279

56. Steph Company’s direct labor costs for the month of January 2006 were as follows: Actual direct labor hours 20,000 Standard direct labor hours 21,000 Direct labor rate variance – unfavorable P 3,000 Total payroll P126,000 What was direct labor efficiency variance? A. P6,000 favorable. C. P6,300 favorable. B. P6,150 favorable. D. P6,450 favorable.

(aicpa)

56. B ? The direct labor efficiency variance.  Direct labor efficiency variance is the difference in hours multiplied by standard rate. The standard rate is not given and should first be determines as follows: Actual labor rate (P126,000 / 20,000 hrs.) P6.30 - Unfavorable rate variance (P3,000 / 20,000 hrs.) 0.15 UF Standard labor rate P6.15 The labor efficiency variance shall be P6,150 favorable. That is, 1,000 hours favorable (i.e., 20,000 hrs. – 21,000 hrs.) times P6.15 standard labor rate. 57. For the month of April, Thorp Company’s records disclosed the following data relating to direct labor: Actual cost P 10,000 Rate variance 1,000 favorable Efficiency variance 1,500 unfavorable Standard cost P 9,500 For the month of April, actual direct labor hours amounted to 2,000. In April, Thorp’s standard direct labor rate per hour was A. P5.50 C. P4.75 B. P5.00 D. P4.50 (aicpa) 57. A ? The standard direct labor rate per hour.  Standard labor rate may be computed as follows: Actual labor rate (P10,000 / 2,000 hrs.) + Change in labor rate (P1,000 /2,000 hrs.) Standard labor rate

P 5.00 0.50 F P5.50

Alternatively, the standard labor rate variance may be derived as follows: DL Rate variance = (AR – SR) x Actual hours (P1,000) F = (P5.00 – SR) x 2,000 hrs. (P1,000) = 10,000 – 2,000SR 2,000SR = 10,000 + 1,000 SR = 11,000 / 2,000 = P5.50

58. Mola Company manufactures one product with a standard direct labor cost of 4 hours at P12.00 per hour. During June 1,000 units were produced using 4,100 hours at P12.20 per hour. The unfavorable direct labor efficiency variance was

Chapter 7 A. P1,220 B. P1,200

Standard costing and variance analysis C. P 820 D. P 400

280

(aicpa)

58. B ? The direct labor efficiency variance.  Direct labor efficiency variance is the difference in actual and standard labor hours, multiplied by the standard labor rate. The standard time is 4,000 hours (i.e., 1,000 units x 4 hours). The direct labor efficiency variance is 100 hrs.,unfavorable (i.e, 4,100 hrs. – 4,000 hrs.). The total direct labor efficiency variance is P1,200, unfavorable (i.e., 100 hrs-unfavorable x P12.00). 59. Information on Hanley’s direct labor costs for the month of January is as follows: Actual direct labor rate P 7.50 Standard direct labor hour allowed 11,000 Actual direct labor hours 10,000 Direct labor rate variance-favorable P 5,500 The standard direct labor rate in January was A. P6.95 C. P8.00 B. P7.00 D. P8.05 (aicpa) 59. D ? The standard direct labor rate.  The standard direct labor rate may be determined from actual direct labor rate with adjustment on labor rate variance, as follows: Actual direct labor rate P7.50 + Change labor rate variance (P5,500/10,000) 0.55 F Standard direct labor rate P8.05 The standard direct labor rate variance may also be derived from the labor rate variance analysis as follows: (P5,500) F = (P7.50 – SR) x 10,000 hrs. (P5,500) = 75,000 – 10,000SR SR = 80,500/10,000 = P8.05 60. Tub Company uses a standard cost system. The direct labor for product B for the month of October: Standard hours allowed for actual production Actual rate paid per hour Standard rate per hour Labor efficiency variance What were the actual hours worked? A. 1,800 C. 2,190 B. 1,810 D. 2,200

following information pertains to 2,000 P 8.40 P 8.00 P1,600 U (aicpa)

60. D ? The actual hours worked.  The actual hours worked may be determined from the standard hours with adjustment for labor efficiency variance, as follows:

Chapter 7

Standard costing and variance analysis

Equation method: Standard hours + Change in labor hours (P1,600/P8.00) Actual hours worked

281

2,000 200 UF 2,200

The actual hours worked may also be derived from the labor efficiency variance analysis, as follows: P1,600 UF = (AH – 2,000) x P8.00 P1,600 = 8AH – 16,000 AH = 17,600/8 = 2,200

61. The following direct labor information pertain to the manufacture of product Celeste: Time required to make one unit 2 DLH Number of direct workers 50 Number of productive hours per week, per worker 40 Weekly wages per worker P500 Worker’s benefits treated as direct labor costs 20% of wages What is the standard direct labor cost per unit of product Celeste? A. P30 C. P15 B. P24 D. P12 (aicpa) 61. A ? The standard direct labor cost per unit.  The standard direct labor rate per hour is computed by standard direct labor cost divided by standard units. The standard labor cost per week per worker is P600 (i.e., P500 x 120%), to include the 20% workers’ benefits. The standard units per worker per week is 20 units (i.e., 40 hrs./2 units per hour). Therefore, the standard labor rate per unit is P30 (i.e., P600/20 units). 62. Sullivan Corporation’s direct labor costs for the month of March were as follows: Standard direct labor hours 42,000 Actual direct labor hours 40,000 Direct labor rate variance-favorable P8,400 Standard direct labor rate per hour P6.30 What was Sullivan’s total direct labor payroll for the month of March? A. P243,600 C. P264,600 B. P252,000 D. P260,400 (aicpa) 62. A ? The total direct labor payroll for the month.  Actual payroll is equal to actual labor hours multiplied by actual labor rate. The actual labor hours are already given. The actual labor rate may be computed as follows: Standard labor rate per hour P6.30 - Change in labor rate (P8,400/40,000 hrs.) 0.21 F Actual labor rate per hour P6.09 Given the actual labor rate per hour, the actual payroll shall be P243,600 (i.e., 40,000 hrs. x P6.09).

Chapter 7

Standard costing and variance analysis

282

Overhead cost variances 63. Under the two-variance method for analyzing factory overhead, the difference between the actual factory overhead and the factory overhead applied to production is the A. Controllable variance. C. Efficiency variance. B. Net overhead variance. D. Volume variance. (aicpa) 63. B ? The name for the difference between actual factory overhead and applied factory overhead.  Applied factory overhead is used to mean the standard factory overhead. The difference between actual factory overhead and standard factory overhead is the net overhead variance. Choice-letter “b” is correct. Choice-letter “a” is incorrect because controllable variance is the difference between actual factory overhead and budgeted overhead based on standard hours. Choice-letter “c” is incorrect because efficiency variance pertains to the difference in actual hours and standard hours. Choice-letter “d” is incorrect because volume variance refers to the difference in normal capacity (or budgeted capacity) and standard capacity, times the standard fixed overhead rate. 64. Under the two-variance method for analyzing factory overhead, the budget allowance based on standard hours allowed is used in the computation of the Controllable Volume (budget) variance variance A. Yes Yes B. Yes No C. No No D. No Yes (aicpa) 64. A ? The variance that uses the budget allowance on standard hours (BASH) in the 2-way analysis method.  Choice-letter “a” is correct. The BASH is used in the computation of controllable and volume variance. Under the 2-way variance analysis, the overhead variance are as follows: Actual factory overhead Controllable variance Budget allowed on standard hours Volume variance Standard factory overhead 65. Under the three-variance method for analyzing factory overhead, which of the following is used in the computation of the spending variance? Factory overhead Budget allowance applied to Based on standard hours production A. Yes Yes B. Yes No

Chapter 7 C. D.

Standard costing and variance analysis No No

Yes No

283

(aicpa)

65. D ? The one used in the computation of spending variance under the 3-variance method.  Using the 3-way method, spending variance is the difference between actual factory overhead (AFOH) and budgeted allowed on actual hours (BAAH). Applied factory overhead, which is equivalent to standard factory overhead, is not used in the computation of overhead spending variance. 66. Under the three-variance method for analyzing factory overhead, the difference between the actual factory overhead and the factory overhead applied to production is the A. Net overhead variance. C. Efficiency variance. B. Controllable variance. D. Spending variance. (aicpa) 66. A ? The name for the difference between actual factory overhead and applied factory overhead.  Applied factory overhead is used to mean the standard factory overhead. The difference between actual factory overhead and standard factory overhead is the net overhead variance. Choice-letter “a” is correct. Choice-letter “b” is incorrect because controllable variance is the difference between actual factory overhead and budgeted overhead based on standard hours (BASH). Choice-letter “c” is incorrect because efficiency variance pertains to the difference in actual hours and standard hours. Choice-letter “d” is incorrect because spending variance is the difference between actual factory variance and budget allowed on actual hours (BAAH). 67. When using the two-variance method for analyzing factory overhead, the difference between the budget allowance based on standard hours allowed and the factory overhead applied to production is the A. Net overhead variance. C. Volume variance. B. Controllable variance. D. Efficiency variance. (aicpa) 67. C ? The variance for the difference in budget allowed for standard hours (BASH) and applied factory overhead.  The difference between BASH and applied factory overhead (i.e., standard overhead) is the controllable variance. Choice-letter “a”, net overhead variance, is the difference between actual overhead and standard overhead. Choice-letter “b”, controllable variance, is the difference between actual factory overhead and BASH. Choice-letter “d”, efficiency variance, is the difference between actual hours and standard hour used. 68. You used predetermined overhead rates and the resulting variances when compared with the results using the actual rates were substantial. Production data indicated that volumes were lower than the plan by a large difference. This situation can be due to:

Chapter 7 A. B. C. D.

Standard costing and variance analysis

Overhead being substantially composed of fixed costs. Overhead being substantially composed of variable cost. Overhead cost being recorded as planned. Products being simultaneously manufactured in single runs.

284

(rpcpa)

68. A ? An explanation for a substantial overhead variance.  It is stated in the problem that there is a large difference in volume between the actual and the plan data, and at the same time there is a substantial overhead variance. The overhead variance may be a substantial volume variance and the overhead is predominantly fixed in nature. Choice-letter “a” is correct. Volume variance is basically the difference between normal capacity and actual capacity multiplied by the unit fixed overhead rate. Choice-letter “b” is incorrect because if overhead is substantially composed of variable cost, then the difference in volume would not have resulted to substantial overhead variance, assuming all other variables are irrelevant. Choice-letter “c” is incorrect because it would result to a zero variance situation. Choice-letter “d” is incorrect because simultaneous running of products in a single run does not relate to variance analysis. 69. Which one of the following variances is of least significance from behavioral control perspective? A. Unfavorable materials quantity variance amounting to 20% of the quantity allowed for the output attained. B. Unfavorable labor efficiency variance amounting to 10% more than the budgeted hours for the output attained. C. Favorable materials price variance obtained by purchasing raw material from a new vendor. D. Fixed factory overhead volume variance resulting from management decision midway through the fiscal year to reduce its budgeted output by 20%. (cma) 69. D ? The variance that is of least significance from behavioral control perspective.  Behavioral control perspective is relevant to those included in the normal plan under certain well-studied standards. However, once a standard is materially altered on matters beyond the control of operating managers such as economic downturns, labor problems, unexpected technological breakthroughs, geological changes (e.g., climate changes), and change in planned output. The variances resulting from this standards are to be handled by top management and are therefore least likely to impact behavioral control processes. Choice-letter “d” is correct because management decision midway through the fiscal year to reduce its budgeted output by 20% is on the level of top management and not in the behavioral aspects of operational officers and personnel. Choice-letters “a”, “b”, and “c” are matters that have behavioral impact on managing people. 70. Which of the following standard costing variances would be least controllable by a production supervisor?

Chapter 7

Standard costing and variance analysis

A. Overhead volume. B. Overhead efficiency.

C. Labor efficiency. D. Materials usage.

285

(aicpa)

70. A ? An item least controllable by a production supervisor.  Operating or production supervisors have direct control over personnel efficiencies in terms of direct labor hours and quantity of materials consumed. Choice-letters “b”, “c”, and “d” are controllable by the production supervisor and are incorrect answers. Choice-letter “a” is the right answer because overhead volume variance specifically relates to units of production and not labor hours and materials usage. Volume variance is controllable by the production plant manager. 71. Under the three-variance method for analyzing factory overhead, the difference between the actual factory overhead and the budget allowance based on actual input is the A. Efficiency variance. C. Volume variance. B. Spending variance. D. Idle capacity variance. (aicpa) 71. B ? The variance that measures the difference between actual factory overhead and budget allowance on actual input.  Spending variance is the difference between actual factory overhead and budget allowance based on actual hours. Choice-letter “a” is incorrect because efficiency variance is the difference in actual hours and standard hours. Choice-letter “c” is incorrect because volume variance is the difference in normal hours and standard hours. Choice-letter “d” is incorrect because idle capacity variance is the difference in normal hours and actual hours used. 72. A spending variance for variable factory overhead based on direct labor hours is the difference between actual variable factory overhead and the variable factory overhead that should have been incurred for the actual hours worked. This variance results from A. Price and quantity differences for factory overhead costs. B. Price differences for factory overhead costs. C. Quantity differences for factory overhead costs. D. Differences caused by variations in production volume. (aicpa) 72. B ? The reason for the occurrence of variable overhead spending variance.  Variable OH Spending variance (or rate variance) is the difference between actual variable factory overhead and budgeted variable overhead on actual hours worked (BAAH). This variance purely relates to the amount of money spent, not in hours worked and quantity produced Choice-letters “a” and “c” are incorrect because spending variance is not concerned on quantity differences. Choice-letter “d” is incorrect because spending variance is not concerned with output variance (volume variance).

Chapter 7

Standard costing and variance analysis

286

73. Variable factory overhead is applied on the basis of standard direct labor hours. If for a given period, the direct labor efficiency variance is unfavorable, the variable factory overhead efficiency variance will be A. B. C. D.

Favorable. Unfavorable. Zero. The same amount as the labor efficiency variance.

(cma)

73. B ? The effect of an unfavorable labor efficiency variance to variable overhead efficiency variance.  Efficiency variance refers to actual hours less standard hours. The same actual and standard hours are used in determining efficiency variances both in labor and overhead. Therefore, unfavorable efficiency variance has the same effect in labor as that of overhead. Choice-letter “b” is correct. Choice-letter ‘a” is incorrect because the labor efficiency is unfavorable, and therefore the overhead efficiency variance should also be unfavorable. Choice-letter ”c” is incorrect because there is an unfavorable efficiency variance. Choice-letter “d” is incorrect because efficiency variance differs from labor and overhead because of the difference in their applicable standard rates. 74. If factory overhead is applied on the basis of units of output, the variable factory overhead efficiency variance will be A. Zero. B. Favorable, if output exceeds the budgeted level. C. Unfavorable, if output is less than the budgeted level D. A function on the direct labor efficiency variance. (cma) 74. A ? The effect to variable factory overhead efficiency variance if the overhead rate is based on units of output.  Efficiency variance is based on hours (i.e., time). If the overhead rates are based on units of output and not on time, the efficiency variance cannot be recognized. Choiceletter “a” is correct. Choice-letters “b”, “c”, and “d” are already irrelevant choices because the fundamental information of overhead rate on hour is not given. 75. Under the two-variance method for analyzing factory overhead, which of the following is used in the computation of the controllable (budget) variance? Budget allowance Budget allowance based on based on actual hours standard hours A. Yes Yes B. Yes No C. No No D. No Yes (aicpa)

Chapter 7

Standard costing and variance analysis

287

75. D ? The one used in the computation of controllable variance using a 2-variance method.  The one referred to here is the controllable variance, being a more basic variance than the budget variance. Controllable variance is the difference between actual factory overhead and budget allowed on standard hours. The budget allowed on actual hours is used in the computation of spending (or budget) variance. Choiceletter “d” is correct. 76. Under the three-variance method for analyzing factory overhead, which of the following is used in the computation of the spending variance? Actual Factory Budgeted Allowance Overhead Based on Actual Input A. No Yes B. No No C. Yes No D. Yes Yes (aicpa) 76. D ? The item(s) used in determining spending variance.  Spending (or budget) variance is actual factory overhead less budget allowance on actual hours. Choice-letter “d” is correct. 77. Differences in product costs resulting from the application of actual overhead rates rather than predetermined overhead rates could be immaterial if A. Production is not stable. B. Fixed factory overhead is a significant cost. C. Several products are produced simultaneously. D. Overhead is composed only of variable costs. (aicpa) 77. D ? A situation where differences in product costs resulting from the application of actual overhead rates rather than predetermined overhead rates are immaterial.  Differences in product costs computed under varying production level would be immaterial if the overhead is composed only of variable costs. This is because variable cost rate is expected to be constant over the relevant range of production levels. Hence, the correct answer is choice-letter “d”. Choice-letters “a” and “b” are incorrect because if production is not stable and total overhead includes fixed overhead or the fixed overhead is a significant component of the total overhead cost, the differences in overhead costs between actual capacity and predetermined capacity would be material. Choice-letter “c” is incorrect because product mix has an impact on the amount of actual and estimated overhead. 78. The variance in an absorption costing system that measures the departure from the denominator level of activity that was used to set the fixed overhead rate is the A. Spending variance. C. Sales volume variance. B. Efficiency variance. D. Production volume variance. (cma)

Chapter 7

Standard costing and variance analysis

288

78. D ? The variance in absorption costing that measures the departure from normal capacity.  Normal capacity is the level of activity used in establishing the fixed overhead rate. Any deviation from the normal capacity (or normal volume) is described as the volume variance, and is related to the fixed overhead rate. Choice-letters ‘a”, “b”, are not related to normal capacity. Choice-letter “c” is incorrect because sales volume variance is not related to fixed overhead rate. Questions 79 through 80 are based on the following information. Dori Casting is a job-order shop that uses a full-absorption, standard cost system to account for its production costs. The overhead costs are applied on a direct-labor-hour basis. 79. Dori’s choice of production volume as a denominator for calculating its factory overhead rate has A. An effect on the variable factory overhead rate for applying costs to production. B. No effect on the fixed factory overhead budget variance. C. No effect on the fixed factory overhead production volume variance. D. No effect on the overall (net) fixed factory overhead variance. (cma) 79. B ? The effect of using production volume as the denominator in calculating factory overhead rate.  A choice in production volume used as a denominator in factory overhead has no effect on budget variance. A budget variance is the difference between the actual overhead and the budgeted overhead in terms of peso amount and not in relation to the level of production. Choice-letter “a” is incorrect because determining the unit variable overhead rate on a particular production level would be the same regardless of changes in production capacity. Variable rate is constant per unit or per hour. Choice-letters “c” and “d” are incorrect because the choice of a denominator in determining fixed overhead rate has an impact on volume variance and even in the overall fixed factory overhead variance. 80. A production volume variance will exist for Dori in a month when A. Production volume differs from sales volume. B. Actual direct labor hours differ from standard allowed direct labor hours. C. The fixed overhead applied on the basis of standard allowed direct labor hours differs from actual fixed factory overhead. D. The fixed overhead applied on the basis of standard allowed direct labor hours differs from the budgeted fixed factory overhead (cma) 80. D ? A situation where production volume variance exists.  Volume variance is the difference in normal capacity (or budgeted capacity, if normal capacity is not given) and standard capacity, multiplied by fixed overhead rate. Choice-letter “a” is incorrect because the change in production and sales is the change in inventory. Choice-letter “b” is incorrect because it refers to efficiency

Chapter 7

Standard costing and variance analysis

289

variance. Choice-letter “c” is incorrect because the computation results to controllable variance. 81. The amount of fixed factory overhead that Dori will apply to finished production is the A. Actual direct labor hours times the standard fixed factory overhead rate per direct labor hour. B. Standard allowed direct labor hours for the actual units of finished output times the standard fixed factory overhead rate per direct labor hour. C. Standard units of output for the actual direct labor hours worked times the standard fixed factory overhead rate per unit of output. D. Actual fixed factory overhead cost per direct labor hour times the standard direct labor hours. (cma) 81. B ? The standard fixed overhead.  Standard fixed overhead equals standard hours times standard fixed overhead rate per hour. Standard hours are the allowed number of hours that should have been used on the actual units produced. Choice-letter “b” is correct. Choice-letter “a” is incorrect because it results to applied fixed overhead. Choice-letter “c” is incorrect because the standard fixed overhead rate should be multiplied by the standard hours and not by the standard units of output. Choiceletter “d” is incorrect because the result of the computation does not give any relevant information. 82. The fixed factory overhead application rate is a function of a predetermined activity level. If standard hours allowed for good output equal this predetermined activity level for a given period, the volume variance will be A. Zero. B. Favorable. C. Unfavorable. D. Either favorable or unfavorable, depending on the budgeted overhead. (aicpa) 82. A ? The amount of volume variance.  Volume variance is the difference in normal capacity and standard capacity. If the normal capacity is not available, the budgeted capacity is used. The predetermined activity level refers to either normal capacity or budgeted capacity. If the standard hours on actual output are equal to the normal capacity, then there is no volume variance. 83. During the current year, a department’s three-variance factory overhead standard costing system reported unfavorable spending and volume variances. The activity level selected for allocating factory overhead to the product was based on 80% of practical capacity. If 100% of practical capacity had been selected instead, how would the reported unfavorable spending and volume variances have been affected? Spending Variance Volume Variance A. Increased Unchanged B. Increased Increased

Chapter 7 C. D.

Unchanged Unchanged

Standard costing and variance analysis Increased Unchanged

290

(aicpa)

83. C ? The effects to spending and volume variances if the activity level is increased from 80% to 100%.  Changes in volume do not affect spending variance because this variance refers to the difference between the actual amount spent and the amount that should have been spent based on actual hours. Spending variance measures the change in peso and not in hours or units. A change in capacity affects volume variance. Specifically, an increase in capacity base would result to a higher normal capacity and would further aggravate the unfavorable volume variance. Choice-letter “c” is correct. 84. The following data are presented: Budgeted Actual Production in units 50,000 55,000 Manufacturing overhead P750,000 P800,000 Sales in units No data 47,000 No beginning inventories The underapplied or overapplied overhead is: A. P25,000 underapplied. C. P75,000 overapplied. B. P25,000 overapplied. D. P75,000 underapplied.

(rpcpa)

84. B ? The underapplied or overapplied overhead.  The underapplied or overapplied is the difference between actual factory overhead and applied factory overhead. Applied FOH is actual units times standard overhead rate. The standard overhead rate is P15 (that is, P750,000 budgeted overhead divided by 50,000 budgeted units). The under (over) applied overhead is: Actual factory overhead P800,000 Applied factory overhead (55,000 units x P15) 825,000 Overapplied factory overhead (P 25,000) F 85. Universal Company uses a standard cost system and prepared the following budgeted amounts at normal capacity for the month of January 2006: Direct labor hours 24,000 Variable factory overhead P 48,000 Fixed factory overhead P108,000 Total factory overhead per direct labor hour P 6.50 Actual data for January 2006 were as follows: Direct labor hours worked 22,000 Total factory overhead P147,000 Standard direct labor hours allowed for capacity attained 21,000 Using the two-way analysis of overhead variances, what is the budget (controllable) variance for January 2006? A. P 3,000 favorable. C. P 9,000 favorable. B. P 5,000 favorable. D. P10,500 unfavorable. (aicpa)

Chapter 7

Standard costing and variance analysis

291

85. A ? The amount of controllable variance.  Controllable variance is the difference between actual factory overhead and budget allowed on standard allowance. The standard fixed overhead rate per hour is P4.50 (i.e., P108,000/24,000 hrs.). And the standard variable overhead rate per hour shall be P2.00 (i.e., P6.50 – P4.50). The determination and segregation of overhead rates are important in overhead variance analysis. The controllable variance is determined as follows: Actual factory overhead P147,000 - Budget allowed on standard hours Fixed overhead P108,000 Variable overhead (21,000 hrs. x P2.00 42,000 150,000 Controllable overhead variance P( 3,000) F The controllable variance is favorable because actual overhead is less than the budgeted overhead. 86. Foundation Company uses a standard cost system For the month of April 2006, total overhead is budgeted at P80,000 based on the normal capacity of 20,000 direct labor hours. At standard each unit of finished product requires 2 direct labor hours. The following data are available for the April 2006 production activity: Equivalent units of production 9,500 Direct labor hours worked 19,500 Actual total overhead incurred P79,500 What amount should Foundation credit to the applied factory overhead account for the month of April 2006? A. P76,000 C. P79,500 B. P78,000 D. P80,000 (aicpa) 86. A ? The amount of applied factory overhead.  Applied factory overhead (or standard factory overhead) is standard hours multiplied by standard overhead rate (Standard FOH = SH x SR). The standard hour is 2 DLH per unit. The total standard hours shall be 19,000 (i.e., 9,500 units x 2 DLH per hr.). Take note that we use the equivalent production in the determination of the total standard hours. The standard overhead rate is P4.00 per hour (i.e, P80,000/20,000 DLH). Therefore, the standard factory overhead shall be P76,000 (i.e., 19,000 hrs. x P4 per DLH). 87. Hope Company uses a flexible budget system and prepared the following information for 2006: Normal Maximum Capacity Capacity Percent of capacity 80% 100% Direct labor hours 32,000 40,000 Variable factory overhead P 64,000 P 80,000

Chapter 7

Standard costing and variance analysis

Fixed factory overhead Total factory overhead rate per direct labor hour

P160,000 P7

292 P160,000 P6

Hope operated at 90% of capacity during 2006. The actual factory overhead for 2006 was P252,000. What was the budget (controllable) overhead variance for the year? A. P36,000 unfavorable. C. P18,000 unfavorable B. P20,000 unfavorable. D. P0. (aicpa) 87. B ? The controllable overhead variance.  Technically, budget variance is different from controllable variance. But since the problem does not specify the method to be used, the 2-way overhead analysis method shall be in effect. Therefore, the controllable variance is to be computed. Controllable variance is the difference between actual factory overhead and budget allowed on standard hours (BASH). Very importantly, the standard overhead rates should be determined. In the given, the fixed overhead rate is P5.00 per hour (i.e., P160,000/32,000 hrs.). Remember, fixed overhead rate is based on normal capacity. The variable overhead rate is computed to be P2.00 per hour (i.e., P64,000 / 32,000) . The standard hours shall be based on the actual capacity which is at 90%, therefore 36,000 hrs. (i.e., 40,000 hrs. x 90%). The controllable variance may now be calculated as follows: Actual factory overhead P252,000 - Budget allowed on standard hours Fixed overhead P160,000 Variable overhead (36,000 hrs. x P2.00) 72,000 232,000 Controllable variance P 20,000 UF 88. The following information is available from the Faith Company: Actual factory overhead P15,000 Fixed overhead expenses, actual P7,200 Fixed overhead expenses, budgeted P7,000 Actual hours 3,500 Standard hours 3,800 Variable overhead rate per DLH P2.50 Assuming that Faith uses a three-way analysis of overhead variances, what is the spending variance? A. P750 favorable. C. P950 favorable. B. P750 unfavorable. D. P200 unfavorable. (aicpa) 88. A ? The amount of overhead spending variance.  Overhead spending variance equals the actual factory overhead less budgeted allowance on actual hours, computed as follows: Actual factory overhead P15,000 - Budget allowance on actual hours: Fixed overhead P7,000 Variable overhead (3,500 hrs. x P2.50/hr.) 8,750 15,750 Overhead spending variance P ( 750) F

Chapter 7

Standard costing and variance analysis

293

Questions 88 and 89 are based on the following data: Based on a month’s normal volume of 50,000 units (100,000 direct labor hours), Raff’s standard cost system contains the following overhead costs: Variable P6 per unit Fixed 8 per unit The following information pertains to the month of March 2006: Units actually produced 38,000 Actual direct labor hours worked 80,000 Actual overhead incurred: Variable P250,000 Fixed 384,000 88. For March 2006, the unfavorable variable overhead spending variance was A. P 6,000 C. P12,000 B. P10,000 D. P22,000

(aicpa)

89. B ? The variable overhead spending variance.  Variable overhead spending variance is actual variable overhead less variable overhead based on actual hours. The standard hours per unit is 2 hours (i.e., 100,000 DLH/50,000 units). The standard variable overhead rate per hour is P3.00 (i.e., P6.00/2 hrs. per unit). With these, the variable overhead spending variance is computed as follows: Actual variable overhead P250,000 - Variable overhead on actual hours (80,000 hrs. x P3.00 per hr.) 240,000 Variable overhead spending variance P 10,000 UF 90.For March 2006, the fixed overhead volume variance was A. P96,000 unfavorable. C. P80,000 unfavorable. B. P96,000 favorable. D. P80,000 favorable.

(aicpa)

90. A ? The fixed overhead volume variance.  Volume variance is the difference between normal capacity and standard capacity. Capacity is better expressed in terms of hours inasmuch as standard overhead rates are also expressed in terms of hours. The standard hours for 38,000 units are 76,000 hours (i.e, 38,000 units x 2 hrs. per unit). The volume variance is determined as follows: Normal capacity 100,000 hrs. - Standard capacity 76,000 Volume variance in hours 24,000 UF x Fixed overhead rate per hour (P8 per unit / 2 hrs. per unit)P 4.00 per hr. Volume variance P96,000 UF

Chapter 7

Standard costing and variance analysis

294

Alternatively, the volume variance may be calculated by getting the difference between budget allowed on standard hours and standard factory overhead, as follows: Budget allowed on standard hours Fixed (50,000 units x P8 per unit) P400,000 Variable (76,000 hrs. x P3 per hr.) 228,000 P628,000 - Standard factory overhead (76,000 hrs. x P7 per hr.) 532,000 Volume variance P 96,000 UF The total standard overhead rate is P7.00 per hour (P4.00 per hour for fixed overhead and P3.00 per hour for variable overhead). 91. Union Company uses a standard cost accounting system. The following factory overhead and production data are available for August: Standard fixed overhead rate per DLH P1 Standard variable overhead rate per DLH P4 Budgeted monthly DLH 40,000 Actual DLH worked 39,500 Standard DLH allowed for actual production 39,000 Overall overhead variance-favorable P2,000 The applied factory overhead for August should be A. P195,000 C. P197,500 B. P197,000 D. P199,500

(aicpa)

91.A ? The applied factory overhead.  The applied factory overhead, the same as the standard factory overhead, equals standard hours multiplied by standard overhead rate per hour. The total standard overhead rate per hour is P5.00 (i.e., P1 + P4). The standard factory overhead, then, is P195,000 (i.e., 39,000 hours x P5 per hour). 92. Peters Company uses a flexible budget system and prepared the following information for the year: Percent of capacity 80% 90% Direct labor hours 24,000 27,000 Variable factory overhead P 48,000 P 54,000 Fixed factory overhead P108,000 P108.000 Total factory overhead rate per DLH P6.50 P6.00 Peters operated at 80% of capacity during the year but applied factory overhead based on the 90% capacity level. Assuming that actual factory overhead was equal to the budgeted amount for the attained capacity, what is the amount of overhead variance for the year? A. P 6,000 overabsorbed. C. P12,000 overabsorbed. B. P 6,000 underabsorbed. D. P12,000 underabsorbed. (aicpa) 92. D ? The amount of overhead variance.

Chapter 7



Standard costing and variance analysis

295

The overhead variance is the difference between actual factory overhead and standard overhead. The actual factory overhead is equal to budgeted overhead at 80% capacity amounting to P156,000 (i.e., P48,000 + P108,000). The standard hours at actual capacity is 24,000 hours. And the standard overhead rate is P6.00, based on a predetermined capacity of 90%. The net overhead variance is: Actual factory overhead P156,000 - Standard factory overhead (24,000 hrs. x P6) 144,000 Net overhead variance - underabsorbed P 12,000 UF The overhead variance is underabsorbed or underapplied. Alternatively, this could also be computed by getting first the fixed overhead rate per hour of P4.00 (i.e., P108,000/27,000 hrs.) multiplied by the volume variance of 3,000 hrs-unfavorable (i.e., 27,000 hrs. – 24,000). Therefore, 3,000 hours times P4.00 equals P12,000 unfavorable volume variance. The total of 27,000 hours is the predetermined capacity while the total of 24,000 hours is the standard capacity. There is no controllable variance because the actual factory overhead equals the budgeted amount for the attained capacity.

93. New Technology Company uses a predetermined factory overhead application rate based on direct labor cost. For the year ended December 31, Neil’s budgeted factory overhead was P600,000, based on budgeted volume of 50,000 direct labor hours, at a standard direct labor rate of P6 per hour. Actual factory overhead amounted to P620,000, with actual direct labor cost P325,000. For the year, overapplied factory overhead was A. P20,000 C. P30,000 B. P25,000 D. P50,000 (aicpa) 93.C ? The amount of overapplied factory overhead.  Underapplied or overapplied factory overhead is the difference between actual factory overhead and standard factory overhead. The standard direct labor is P300,000 (i.e., 50,000 DLH x P5 per hour). And the factory overhead application rate is 200% based on direct labor cost (i.e., P600,000/P300,000). The overapplied factory overhead is P30,000, computed as follows: Actual factory overhead P620,000 - Applied factory overhead (P325,000 x 200%) 650,000 Overapplied factory overhead P(30,000) Questions 94 and 95 are based on the following information: The following information relates to a given department of Herman Company for the fourth quarter of 2006: Actual total overhead (fixed plus variable) P178,500 Budget formula P110,000 plus P0.50 /hr. Total overhead application rate P1.50/hr. Spending variance P 8,000 unfavorable Volume variance P 5,000 favorable

Chapter 7

Standard costing and variance analysis

296

The total overhead variance is divided into three variances – spending, efficiency, and volume. 94. What were the actual hours worked in the department during the quarter? A. 110,000 C. 137,000 B. 121,000 D. 153,000 94. B ? The actual hours worked during the period.  Actual hours worked is in the computation of spending variance, where, spending variance is actual factory overhead less budgeted overhead based on actual hours (BAAH). Since, the spending variance is already given, the BAAH may be computed and consequently the actual hours, as follows: Actual factory overhead P178,500 - Unfavorable spending variance 8,000 F Budget allowed on actual hours 170,500 - Fixed factory overhead 110,000 Variable overhead at actual hours 60,500 / Standard variable overhead rate P 0.50 per hr. Actual hours 121,000 hrs. 95. What were the standard hours allowed for good output in this department during the quarter? A. 105,000 C. 110,000 B. 106,667 D. 115,000 (aicpa) 95. D ? The standard hours for good output.  Standard hours are used in the computation of volume variance, which is the difference between budgeted allowed on standard hours (BASH) and standard factory overhead. Since the volume variance is given at P5,000 favorable, the standard hours may be determined, as follows: If x = standard hours then: BASH = P110,000 + (0.50x) Standard OH = 1.50x where: BASH – Standard OH = (P5,000) F therefore: (P110,000 + 0.50x) – 1.50x = (5,000) 110,000 – 1.00x = (5,000) 110,000 + P5,000 = 1.00x X = 115,000 hrs. Questions 96 and 97 are based on the following information. Tiny Tweety Corporation had the following activity relating to its fixed and variable overhead for the month of July: Actual costs Fixed overhead P 120,000 Variable overhead 80,000

Chapter 7

Standard costing and variance analysis

297

Flexible budget (Standard input allowed for actual output achieved x the budgeted rate) Variable overhead 90,000 Applied (Standard input allowed for actual output achieved x the budgeted rate) Fixed overhead 125,000 Variable overhead spending variance 2,000 F Production volume variance 5,000 U 96. If the budgeted rate for applying variable factory overhead was P20 per direct labor hour, how efficient or inefficient was Tiny Tweety Corporation in terms of using direct labor hours as an activity base? A. 100 direct labor hours inefficient. B. 100 direct labor hours inefficient. C. 400 direct labor hours inefficient. D. 400 direct labor hours efficient. (cma) 96. D ? The direct labor efficiency variance.  Efficiency variance is the difference between actual hours and standard hours. Variable overhead efficiency variance and variable overhead spending variance comprise the total of variable overhead variance. Since the actual variable overhead and flexible overhead are given, the total variable overhead variance may be determined. From this variance, we deduct the variable overhead spending variance to get the variable efficiency variance, as follows: Actual variable factory overhead P80,000 - Budgeted variable overhead on standard capacity 90,000 Total variable overhead variance (10,000) F - Variable overhead spending variance ( 2,000) F Variable efficiency variance – in pesos ( 8,000) F / Standard variable overhead rate P 20 per hour Efficiency variance - in hours (400) F A favorable efficiency variance indicates efficient use of manpower hours. Hence, choice-letter “d” is correct. 97. The fixed factory overhead efficiency variance is A. P3,000 favorable C. P5,000 favorable B. P3,000 unfavorable D. Never a meaningful variance.

(cma)

97. D ? The fixed factory overhead efficiency variance.  Fixed overhead efficiency variance is the difference in actual hours and standard hours, multiplied by the fixed overhead rate per hour. Fixed overhead is related to level of output in units and not to level of hours used. Although, the fixed overhead efficiency variance may be computed, it does not convey relevant meaning in relation to managing fixed overhead. Therefore, choice-letter “d” is correct.

Chapter 7

Standard costing and variance analysis

298

Questions 98 through 102 are based on the following information. Ninja Company manufactures a line of products distributed nationally through wholesaler. Presented below are planned manufacturing data for 2006 and actual data for November 2006. The company applies overhead based on planned machine hours using a predetermined annual rate. Fixed factory overhead Variable factory overhead Direct labor hours Machine hours

2006 Planning Data Annual November P1,200,000 P100,000 2,400,000 220,000 48,000 4,000 240,000 22,000

Data for November 2006 Direct labor hours (actual) 4,200 Direct labor hours (plan based on output) 4,000 Machine hours (actual) 21,600 Machine hours (plan based on output) 21,000 Fixed factory overhead P101,200 Variable factory overhead P214,000

98. The predetermined factory overhead application rate for Ninja Company for 2006 is A. P 5.00 C. P50.00 B. P25.00 D. P15.00 (cma) 98. D ? The predetermined factory overhead application rate.  The company applies overhead on planned machine hours. The standard overhead rates are: Fixed overhead (P1,200,000/240,000 MH) P 5.00 / MH Variable overhead (P2,400,000/240,000 MH) 10.00 / MH Total overhead rate P15.00 / MH 99. The total amount of factory overhead applied to production for November 2006 was A. P315,200 C. P300,000 B. P315,000 D. P324,000 (cma) 99. B ? The amount of factory overhead applied to production.  The amount of factory overhead applied to production is the standard factory overhead. It is equal to standard hours times standard rate per hour. The standard machine hours in November is 21,000 MH. Therefore, the standard factory overhead is P315,000 (i.e., 21,000 MH x P15 per MH). 100.The amount of over-or-underapplied variable factory overhead for November was A. P6,000 overapplied. C. P20,000 overapplied. B. P4,000 underapplied.

D. P6,000 underapplied.

(cma)

100.B ? The amount of variable overhead variance in November.  Variable overhead variance is actual variable overhead less budgeted variable on standard hours, as follows:

Chapter 7

Standard costing and variance analysis

299

Actual variable overhead P214,000 - Budgeted variable overhead on standard hours (21,000 MH x P10 per MH) 210,000 Variable overhead variance P 4,000 UF An unfavorable overhead variance means an underapplied variance. 101. The variable factory overhead spending variance for November 2006 was A. P2,000 favorable. C. P14,000 unfavorable. B. P6,000 favorable. D. P6,000 unfavorable.

(cma)

101.A ? The variable overhead spending variance in November.  Variable overhead spending variance is actual overhead less budgeted variable overhead based on actual hours, as follows: Actual variable overhead P214,000 - Budgeted variable overhead on actual hours (21,600 MH x P10 per MH) 216,000 Variable overhead spending variance P (2,000) F 102. The fixed factory overhead volume variance for November 2006 was. A. P1,200 unfavorable. C. P5,000 favorable. B. P5,000 unfavorable. D. P1,200 favorable.

(cma)

102.D ? The fixed overhead volume variance.  Volume variance, which relates to fixed overhead, is computed by getting the difference between budget allowed on standard hours and standard factory overhead, as follows: Budget allowed on standard hours Fixed overhead P100,000 Variable overhead (21,000 hrs x P10) 210,000 P310,000 - Standard factory overhead (21,000 hrs. x P15) 315,000 Volume variance P( 5,000) F Alternatively, volume variance may be computed as follows: Normal capacity (240,000 MH/12 mos.) 20,000 MH. - Standard capacity 21,000 Volume variance -in hours ( 1,000) F x Fixed overhead rate per MH P 5.00 Volume variance - in pesos P(5,000) F 103.Selo Imports uses flexible budgeting for the control of costs. The company’s annual master budget includes P324,000 for fixed production supervisory salaries at a volume of 180,000 units. Supervisory salaries are expected to be incurred uniformly through the year. During September, 15,750 units were purchased and production supervisory salaries incurred were P28,000. A performance report for September should reflect a budget variance of A. P 350 F. C. P1,000 U.

Chapter 7 B. P 350 U.

Standard costing and variance analysis D. P1,000 F.

300 (cma)

103.C ? The amount of budget variance.  Budget variance is the difference between actual factory overhead and budget allowed on actual hours. Or, budget variance is actual overhead less flexible budget. Of importance is the information that supervisory salaries are expected to be incurred uniformly through the year. The budget variance is determined, as follows: Actual salaries incurred P28,000 - Budgeted salaries based on actual output (P324,000/12 mos.) 27,000 Budget variance P( 1,000) F 104.Samson Company’s master budget shows straight-line depreciation on factory equipment of P258,000. The master budget was prepared at an annual production volume of 103,200 units of product. This production volume is expected to occur uniformly throughout the year. During September, Simson produced 8,170 units of product, and the accounts reflected actual depreciation on factory machinery of P20,500. Simson controls manufacturing costs with a flexible budget. The flexible budget amount for depreciation on factory machinery for September should be A. P19,875 C. P20,500 B. P20,425 D. P21,500 (cma) 104.D ? The flexible amount for depreciation on factory machinery for September.  The production volume is expected to occur uniformly throughout the year. Therefore, the budgeted depreciation expense in September shall P21,500 (i.e., P258,000 / 12 mos.) 105.Mars Company ends the month with a volume variance of P6,360 unfavorable. If budgeted fixed overhead was P480,000, overhead was applied on the basis of 32,000 budgeted machine hours, and budgeted variable factory overhead was P170,000, what was the actual hours (AH) for the month? A. 32,424 C. 31,687 B. 32,000 D. 31,576 (ing) 105.D ? The actual machine hours for the month.  The overhead rate is P15 (i.e., P480,000/32,000 hrs.). Note that overhead is applied on actual hours. The applied overhead is determined using the volume variance, as follows: Budgeted overhead P480,000 - Volume variance – unfavorable 6,360 UF Applied overhead on actual 473,640 / Standard overhead rate P 15 Actual hours 31,576

Chapter 7

Standard costing and variance analysis

301

Variance disposition 106.How should a usage variance that is significant in amount be treated at the end of an accounting period? A. Reported as a deferred charge or credit. B. Allocated among work-in-process inventory, finished goods inventory, and cost of goods sold. C. Charged or credited to cost of goods manufactured. D. Allocated among cost of goods manufactured, finished goods inventory, and cost of goods sold. (aicpa) 106.B ? The treatment of a significant usage variance at the end of an accounting period.  Material variances, including usage variance, are allocated to cost of goods sold, work-in-process inventories, and finished goods inventories to reflect a more reasonable amount of costs charged against revenue and the true value of inventories. Choice-letter “a” is incorrect because a significant potion of variances must be allocated to cost of goods sold and only those cost variances allocated to cost of goods sold are defined. Choice-letters “c” and “d” are incorrect because variances are proper to be charged to cost of goods sold rather than to cost of goods manufactured. 107.Standard costing will produce the same income before extraordinary items as actual when standard cost variances are assigned to A. Work-in-process and finished goods inventories. B. An income or expense account. C. Cost of goods sold and inventories. D. Cost of goods sold. (aicpa) 107.C ? The allocation of cost variances to make standard costing equal to actual.  Cost variances are disposed depending on their effects. If the variance is normal, it is closed to cost of goods sold. If the variance is material, it is allocated to cost of goods sold and inventories. One of the major purposes of closing variances is to bring the standard cost figures equal to actual costs. This is appropriately done by allocating variances to cost of goods sold and inventories. Choice-letter “a” is incorrect because a big portion of variances should also be closed to cost of goods sold. Choice-letter “b” is incorrect because allocating variances to income and expense accounts would not reflect the true relations and intentions of the variances. Choice-letter “d” is incorrect because a portion of the variances should be closed to inventories. 108.At the end of its fiscal year. Graham Company had several substantial variances from standard variable manufacturing costs. The one that should be allocated between inventories and cost of sales is the one attributable to A. Additional cost of raw material acquired under a speculative purchase contract. B. A breakdown of equipment

Chapter 7

Standard costing and variance analysis

302

C. Overestimates of production volume for the period resulting from failure to predict an unusual decline in the market for the company’s product. D. Increased labor rates won by the union as a result of a strike. (aicpa) 108.D ? The variance from standard variable manufacturing costs that should be allocated to cost of sales and inventories.  Cost variances are based on standard costs, which are, in turn, based on routinary events. Increase in labor rates are part of doing regular business, routinary, and therefore should be considered a variance to be allocated in the cost of goods sold and inventories. Choice-letter “a” is incorrect because results of transactions resulting from speculations are not covered by standard costs. Choice-letter “b” is incorrect because during a breakdown, variable costs are not normally incurred. Choice-letter “c” is incorrect because it concerns about volume variance, which is related to fixed costs and not to variable costs. 109.What is the normal year-end treatment of immaterial variances recognized in a cost accounting system using standard costs? A. Reclassified as deferred charges until all related production is sold. B. Allocated among cost of goods manufactured and ending work-in-process inventory C. Closed to cost of goods sold in the period in which they arose. D. Capitalized as a cost of ending finished goods inventory. (aicpa) 109.C ? The normal year-end treatment of immaterial cost variances.  Normal or immaterial cost variances are closed to closed to goods sold. Choiceletter “c” is correct. Choice-letter “a” is incorrect because variances are deferred only in proportion to the amounts of inventories and when the amount of the variance is material. Choiceletter “b” is incorrect because only material cost variances are allocated among cost of goods sold and inventories, not even to cost of goods manufactured. Choice-letter “d” is incorrect because only material variances are capitalized as a cost of ending inventory. 110.Which of the following is not an acceptable treatment of factory overhead variances at an interim reporting date? A. Apportion the total only among work-in-process and finished goods inventories on hand at the end of the interim reporting period. B. Apportion the total only between that part of the current period’s production remaining in inventories at the end of the period and that part sold during the period. C. Carry forward the total to be offset by opposite balances in later periods D. Charge or credit the total to the cost of goods sold during the period. (aicpa) 110.A ? The unacceptable treatment of factory overhead variances at an interim reporting data.

Chapter 7

Standard costing and variance analysis

303

 For interim reporting, factory overhead variances could either be deferred or carried forward to the next period, charged or credited to the cost of goods sold, or apportioned among cost of goods sold and inventories. Choice-letters “b”, “c”, and “d” are acceptable treatments of factory overhead variances at an interim reporting. Choice-letter “a” is incorrect because apportioning the factory overhead variances to work-in-process and finished goods inventories is not acceptable because the cost of goods sold shall also be given an allocation in proportion to the number of units sold.  done 

View more...

Comments

Copyright ©2017 KUPDF Inc.
SUPPORT KUPDF